You are on page 1of 91

CHN and OB Question 1 Situation 1: The Department of Health is strengthening its programs regarding maternal and child health

nursing. As a newly appointed community health nurse in town, you are oriented and well verse on these. With regards to Womens Health and Safe Motherhood Project, the following strategies are included to prevent maternal mortality, except one: A B C D Establishment of BEmoNC and CEmoNC networks TBA facilitated home deliveries Improved Family Planning counselling Emphasis on Facility based deliveries

Question 2 The following qualifications reflect possible delivery in the Rural Health Unit, except for one: A B C D Cephalic presentation Adequate pelvis History of ceasarian section delivery Less than five pregnancies

Question 3 You are assigned to provide baby care to the newly delivered healthy baby. What procedure are you going to do first? A B C D Examine the newborn and check for defects, deformities and birth injuries. Provide warmth to the newborn and have a quick check for breathing Clamp and cut the cord after cord pulsations have stopped. Facilitate the bonding between the mother and the newborn through early skin to - skin contact

Question 4 The basis for the improvement in strategies in maternal and neonatal care is on Millennium Development Goals, which are:

A B C D

MDGs 1 & 2 MDGs 2 & 4 MDGs 3 & 5 MDGs 4 & 5

Question 5 The following are the intermediate results that can lower the risk for dying from pregnancy and childbirth with the integration of the MNCHN services in the community: A B C D Every delivery is facility- based and managed by skilled birth attendants Every pregnancy is adequately managed during pre and post partum courses Every pregnancy is wanted, planned and supported Only answers A & C

Question 6 Situation II: You have observed that there are reports of dengue hemorrhagic fever in the barangay. You are to perform community awareness lecture about the said disease. Which is the best preventive measure for dengue hemorrhagic fever? A B C D Frequent fogging in the vicinity to kill mosquitoes Use of mosquito nets and mosquito coils Use of mosquito repellent lotions Cleaning of surroundings and proper disposal of coconut shells, tires, and containers

Question 7 What are the signs and symptoms of DHF assessed as Grade II? A B C D Hermans signs, bone and joint pains, fever with headache, and abdominal pain Hypotension, anorexia, nausea and vomiting, fever, and rapid weak pulse Hermans signs, fever with headache, epistaxis, and melena Narrowing pulse pressure, restlessness, and cold clammy

perspiration Question 8 For planning and implementation during the course of DHF, the following are the necessary nursing considerations: A B C D Educate client to avoid dark colored foods and the use of hard bristled toothbrush, razor and other sharp objects Monitoring the intake and output of client Encouraging client to increase fluid intake All of the above

Question 9 What diagnostic procedure will confirm that the client is having DHF? A B C D Blood smear Urine and Stool examinations Cerebrospinal fluid examination Sputum examination

Question 10 Dengue hemorrhagic fever can be fatal. The following manifestations are present in DHF, except for one. A B C D Thrombocytosis Prolonged bleeding time Thrombocytopenia Positive Rumpel Leade test

Question 11 Situation III: IMCI or the Integrated Management for Childhood Illnesses is an important tool for public health nurses for managing cases of children seeking health care. What should you look for in assessing the childs condition? A B C D Does the child vomit everything? Is the child able to breastfeed or drink? Is the child lethargic or unconscious? Is the child having some breathing difficulties?

Question 12 If the child is having fever for 9 days everyday and has stiff neck but without runny nose, what color health management will it suggest? A B C D Green Yellow Pink Violet

Question 13 In a child reported to having diarrhea, assessment is an essential guide for knowing if the child is experiencing dehydration. What body part should you assess? A B C D Buttocks Abdomen Eyelids Arms

Question 14 Child Isabel is already 12 weeks old. What immunizations should Isabel have received? A B C D BCG, OPV-0, DPT+HIB-1, Hepatitis B1, and OPV-1 BCG, OPV-0, DPT+HIB-1, Hepatitis B1, DPT+HIB-2, Hepatitis B2, OPV-1 and OPV-2 BCG, OPV-0, DPT+HIB-1, Hepatitis B1, OPV-1, and Measles BCG, OPV-0, DPT+HIB-1, and OPV-1

Question 15 In preparing sugar water for the treatment of low blood sugar in a child, includes the following proportion: A B C D 200 ml of clean water plus 2 level tbsp. of sugar 400 ml of clean water plus 4 level tsp of sugar 200 ml of clean water plus 4 level tsp of sugar 200 ml of clean water plus 4 level tbsp of sugar

Question 16 Situation IV: The Aquino Health Agenda (AHA) is focusing on the achievement of Universal Health Care for All Filipinos. The AHA strategic thrusts are the keys for ensuring that all Filipinos especially the poor will receive the benefits of the health reform. These involve: A B C D Attainment of Millennium Development Goals 2 and 3. Financial risk protection through expansion in National Health Insurance Program enrolment and benefit Improved service delivery for health care needs through upgraded quality health care facilities B & C only

Question 17 Last April 2011, the Department of Health together with the LGUs conducted a nationwide campaign on: A B C D Reproductive Health Bill Disaster and Preparedness Measles- Rubella Isang Milyong Sepilyo

Question 18 Which is incorrect regarding the training and deployment of unemployed nurses as RNHeals to the rural area? A B C D To supply the needs of poor Filipino people in far flung areas To contribute to the eradication of poverty and hunger To assist in the promotion of gender and equality To address the proliferation of volunteer nurses

Question 19 According to the AHA, the following instruments are vital in implementing the three strategic thrusts. Which one refers to the access to professional health providers capable in the provision of their health needs at the appropriate level of care? A Service Delivery

B C D

Governance for Health Human Resources for Health Health Information

Question 20 Encouragement of community integration and self- reliance enhancement in the community is valued, too in this health agenda. Which of these statements indicate the correct meaning of community health team? A B C D It is a group of people composed of NGOs and private organizations It is led by the midwife in the priority population areas The rural health physician is the one who directly tracks the eligible population None of the above

Question 21 It refers to the description of disease occurrence which is constantly present in a given area. A B C D Pandemic Sporadic Endemic Epidemic

Question 22 Mang Ernie came to the RHU because he was bitten by their dog. What nursing consideration is your priority? A B C D Administration of Anti- rabies vaccine Provision of dim, quiet and non- stimulating room for the client Assessment of wound for classification, severity, and other signs and symptoms Provision of isolation precautions

Question 23

You are about to provide health teachings about pulmonary tuberculosis in your area of assignment. What diagnostic procedure is usually done early in the morning to confirm PTB? A B C D Chest X- ray Sputum Examination Bronchoscopy All of the above

Question 24 Scarlet fever is a febrile contagious condition. Which laboratory procedure is not included in confirming scarlet fever? A B C D ASO titer Sputum Examination Throat culture Differential count of white blood cells

Question 25 What skin manifestation can be observed to a child having the 3- day measle? A B C D Generalized flushing of the skin Rashes that appears on chest spreading gradually upward and downward Macupapular rashes on the cheeks (slightly elevated) Rose- red papules on the face

CHN AND OB 2 1. May arrives at the health care clinic and tells the nurse that her last menstrual period was 9 weeks ago. She also tells the nurse that a home pregnancy test was positive but she began to have mild cramps and is now having moderate vaginal bleeding. During the physical examination of the client, the nurse notes that May has a dilated cervix. The nurse determines that May is experiencing which type of abortion?

a. Inevitable

b. Incomplete c. Threatened d. Septic

2. Nurse Reese is reviewing the record of a pregnant client for her first prenatal visit. Which of the following data, if noted on the clients record, would alert the nurse that the client is at risk for a spontaneous abortion?

a. Age 36 years b. History of syphilis c. History of genital herpes d. History of diabetes mellitus

3. Nurse Hazel is preparing to care for a client who is newly admitted to the hospital with a possible diagnosis of ectopic pregnancy. Nurse Hazel develops a plan of care for the client and determines that which of the following nursing actions is the priority?

a. Monitoring weight b. Assessing for edema c. Monitoring apical pulse d. Monitoring temperature

4. Nurse Oliver is teaching a diabetic pregnant client about nutrition and insulin needs during pregnancy. The nurse determines that the client understands dietary and insulin needs if the client states that the second half of pregnancy require:

a. Decreased caloric intake b. Increased caloric intake c. Decreased Insulin d. Increase Insulin

5. Nurse Michelle is assessing a 24 year old client with a diagnosis of hydatidiform mole. She is aware that one of the following is unassociated with this condition?

a. Excessive fetal activity. b. Larger than normal uterus for gestational age. c. Vaginal bleeding d. Elevated levels of human chorionic gonadotropin.

6. A pregnant client is receiving magnesium sulfate for severe pregnancy induced hypertension (PIH). The clinical findings that would warrant use of the antidote , calcium gluconate is:

a. Urinary output 90 cc in 2 hours. b. Absent patellar reflexes. c. Rapid respiratory rate above 40/min. d. Rapid rise in blood pressure.

7. During vaginal examination of Janah who is in labor, the presenting part is at station plus two. Nurse, correctly interprets it as:

a. Presenting part is 2 cm above the plane of the ischial spines. b. Biparietal diameter is at the level of the ischial spines. c. Presenting part in 2 cm below the plane of the ischial spines. d. Biparietal diameter is 2 cm above the ischial spines.

8. A pregnant client is receiving oxytocin (Pitocin) for induction of labor. A condition that warrant the nurse in-charge to discontinue I.V. infusion of Pitocin is:

a. Contractions every 1 minutes lasting 70-80 seconds. b. Maternal temperature 101.2 c. Early decelerations in the fetal heart rate. d. Fetal heart rate baseline 140-160 bpm.

9. Calcium gluconate is being administered to a client with pregnancy induced hypertension (PIH). A nursing action that must be initiated as the plan of care throughout injection of the drug is:

a. Ventilator assistance b. CVP readings c. EKG tracings d. Continuous CPR

10. A trial for vaginal delivery after an earlier caesareans, would likely to be given to a gravida, who had:

a. First low transverse cesarean was for active herpes type 2 infections; vaginal culture at 39 weeks pregnancy was positive. b. First and second caesareans were for cephalopelvic disproportion. c. First caesarean through a classic incision as a result of severe fetal distress. d. First low transverse caesarean was for breech position. Fetus in this pregnancy is in a vertex presentation.

11.Nurse Ryan is aware that the best initial approach when trying to take a crying toddlers temperature is:

a. Talk to the mother first and then to the toddler. b. Bring extra help so it can be done quickly. c. Encourage the mother to hold the child. d. Ignore the crying and screaming.

12.Baby Tina a 3 month old infant just had a cleft lip and palate repair. What should the nurse do to prevent trauma to operative site?

a. Avoid touching the suture line, even when cleaning. b. Place the baby in prone position. c. Give the baby a pacifier. d. Place the infants arms in soft elbow restraints.

13. Which action should nurse Marian include in the care plan for a 2 month old with heart failure?

a. Feed the infant when he cries. b. Allow the infant to rest before feeding. c. Bathe the infant and administer medications before feeding. d. Weigh and bathe the infant before feeding.

14.Nurse Hazel is teaching a mother who plans to discontinue breast feeding after 5 months. The nurse should advise her to include which foods in her infants diet?

a. Skim milk and baby food. b. Whole milk and baby food. c. Iron-rich formula only. d. Iron-rich formula and baby food.

15.Mommy Linda is playing with her infant, who is sitting securely alone on the floor of the clinic. The mother hides a toy behind her back and the infant looks for it. The nurse is aware that estimated age of the infant would be:

a. 6 months b. 4 months c. 8 months d. 10 months

16.Which of the following is the most prominent feature of public health nursing?

a. It involves providing home care to sick people who are not confined in the hospital. b. Services are provided free of charge to people within the catchments area. c. The public health nurse functions as part of a team providing a public health nursing services. d. Public health nursing focuses on preventive, not curative, services.

17.When the nurse determines whether resources were maximized in implementing Ligtas Tigdas, she is evaluating

a. Effectiveness b. Efficiency c. Adequacy d. Appropriateness

18.Vangie is a new B.S.N. graduate. She wants to become a Public Health Nurse. Where should she apply?

a. Department of Health b. Provincial Health Office c. Regional Health Office d. Rural Health Unit

19.Tony is aware the Chairman of the Municipal Health Board is:

a. Mayor b. Municipal Health Officer c. Public Health Nurse d. Any qualified physician

20.Myra is the public health nurse in a municipality with a total population of about 20,000. There are 3 rural health midwives among the RHU personnel. How many more midwife items will the RHU need?

a. 1 b. 2 c. 3 d. The RHU does not need any more midwife item.

21.According to Freeman and Heinrich, community health nursing is a developmental service. Which of the following best illustrates this statement?

a. The community health nurse continuously develops himself personally and professionally. b. Health education and community organizing are necessary in providing community health services. c. Community health nursing is intended primarily for health promotion and prevention and treatment of disease.

d. The goal of community health nursing is to provide nursing services to people in their own places of residence.

22.Nurse Tina is aware that the disease declared through Presidential Proclamation No. 4 as a target for eradication in the Philippines is?

a. Poliomyelitis b. Measles c. Rabies d. Neonatal tetanus

23.May knows that the step in community organizing that involves training of potential leaders in the community is:

a. Integration b. Community organization c. Community study d. Core group formation

24.Beth a public health nurse takes an active role in community participation. What is the primary goal of community organizing?

a. To educate the people regarding community health problems b. To mobilize the people to resolve community health problems c. To maximize the communitys resources in dealing with health problems.

d. To maximize the communitys resources in dealing with health problems.

25.Tertiary prevention is needed in which stage of the natural history of disease?

a. Pre-pathogenesis b. Pathogenesis c. Prodromal d. Terminal

26.The nurse is caring for a primigravid client in the labor and delivery area. Which condition would place the client at risk for disseminated intravascular coagulation (DIC)?

a. Intrauterine fetal death. b. Placenta accreta. c. Dysfunctional labor. d. Premature rupture of the membranes.

27.A fullterm client is in labor. Nurse Betty is aware that the fetal heart rate would be:

a. 80 to 100 beats/minute b. 100 to 120 beats/minute c. 120 to 160 beats/minute

d. 160 to 180 beats/minute

28.The skin in the diaper area of a 7 month old infant is excoriated and red. Nurse Hazel should instruct the mother to:

a. Change the diaper more often. b. Apply talc powder with diaper changes. c. Wash the area vigorously with each diaper change. d. Decrease the infants fluid intake to decrease saturating diapers.

29.Nurse Carla knows that the common cardiac anomalies in children with Down Syndrome (tri-somy 21) is: a. Atrial septal defect b. Pulmonic stenosis c. Ventricular septal defect d. Endocardial cushion defect

30.Malou was diagnosed with severe preeclampsia is now receiving I.V. magnesium sulfate. The adverse effects associated with magnesium sulfate is:

a. Anemia b. Decreased urine output c. Hyperreflexia d. Increased respiratory rate

31.A 23 year old client is having her menstrual period every 2 weeks that last for 1 week. This type of menstrual pattern is bets defined by:

a. Menorrhagia b. Metrorrhagia c. Dyspareunia d. Amenorrhea

32.Jannah is admitted to the labor and delivery unit. The critical laboratory result for this client would be:

a. Oxygen saturation b. Iron binding capacity c. Blood typing d. Serum Calcium

33.Nurse Gina is aware that the most common condition found during the secondtrimester of pregnancy is:

a. Metabolic alkalosis b. Respiratory acidosis c. Mastitis d. Physiologic anemia

34.Nurse Lynette is working in the triage area of an emergency department. She sees that several pediatric clients arrive simultaneously. The client who needs to be treated first is:

a. A crying 5 year old child with a laceration on his scalp. b. A 4 year old child with a barking coughs and flushed appearance. c. A 3 year old child with Down syndrome who is pale and asleep in his mothers arms. d. A 2 year old infant with stridorous breath sounds, sitting up in his mothers arms and drooling.

35.Maureen in her third trimester arrives at the emergency room with painless vaginal bleeding. Which of the following conditions is suspected?

a. Placenta previa b. Abruptio placentae c. Premature labor d. Sexually transmitted disease

36.A young child named Richard is suspected of having pinworms. The community nurse collects a stool specimen to confirm the diagnosis. The nurse should schedule the collection of this specimen for:

a. Just before bedtime

b. After the child has been bathe c. Any time during the day d. Early in the morning

37.In doing a childs admission assessment, Nurse Betty should be alert to note which signs or symptoms of chronic lead poisoning?

a. Irritability and seizures b. Dehydration and diarrhea c. Bradycardia and hypotension d. Petechiae and hematuria

38.To evaluate a womans understanding about the use of diaphragm for family planning, Nurse Trish asks her to explain how she will use the appliance. Which response indicates a need for further health teaching?

a. I should check the diaphragm carefully for holes every time I use it b. I may need a different size of diaphragm if I gain or lose weight more than 20 pounds c. The diaphragm must be left in place for atleast 6 hours after intercourse d. I really need to use the diaphragm and jelly most during the middle of my menstrual cycle.

39.Hypoxia is a common complication of laryngotracheobronchitis. Nurse Oliver should frequently assess a child with laryngotracheobronchitis for: a. Drooling

b. Muffled voice c. Restlessness d. Low-grade fever

40.How should Nurse Michelle guide a child who is blind to walk to the playroom?

a. Without touching the child, talk continuously as the child walks down the hall. b. Walk one step ahead, with the childs hand on the nurses elbow. c. Walk slightly behind, gently guiding the child forward. d. Walk next to the child, holding the childs hand.

41.When assessing a newborn diagnosed with ductus arteriosus, Nurse Olivia should expect that the child most likely would have an:

a. Loud, machinery-like murmur. b. Bluish color to the lips. c. Decreased BP reading in the upper extremities d. Increased BP reading in the upper extremities.

42.The reason nurse May keeps the neonate in a neutral thermal environment is that when a newborn becomes too cool, the neonate requires:

a. Less oxygen, and the newborns metabolic rate increases. b. More oxygen, and the newborns metabolic rate decreases.

c. More oxygen, and the newborns metabolic rate increases. d. Less oxygen, and the newborns metabolic rate decreases.

43.Before adding potassium to an infants I.V. line, Nurse Ron must be sure to assess whether this infant has:

a. Stable blood pressure b. Patant fontanelles c. Moros reflex d. Voided

44.Nurse Carla should know that the most common causative factor of dermatitis in infants and younger children is:

a. Baby oil b. Baby lotion c. Laundry detergent d. Powder with cornstarch

45.During tube feeding, how far above an infants stomach should the nurse hold the syringe with formula?

a. 6 inches b. 12 inches

c. 18 inches d. 24 inches

46. In a mothers class, Nurse Lhynnete discussed childhood diseases such as chicken pox. Which of the following statements about chicken pox is correct?

a. The older one gets, the more susceptible he becomes to the complications of chicken pox. b. A single attack of chicken pox will prevent future episodes, including conditions such as shingles. c. To prevent an outbreak in the community, quarantine may be imposed by health authorities. d. Chicken pox vaccine is best given when there is an impending outbreak in the community.

47.Barangay Pinoy had an outbreak of German measles. To prevent congenital rubella, what is the BEST advice that you can give to women in the first trimester of pregnancy in the barangay Pinoy?

a. Advice them on the signs of German measles. b. Avoid crowded places, such as markets and movie houses. c. Consult at the health center where rubella vaccine may be given. d. Consult a physician who may give them rubella immunoglobulin.

48.Myrna a public health nurse knows that to determine possible sources of sexually transmitted infections, the BEST method that may be undertaken is:

a. Contact tracing b. Community survey c. Mass screening tests d. Interview of suspects

49.A 33-year old female client came for consultation at the health center with the chief complaint of fever for a week. Accompanying symptoms were muscle pains and body malaise. A week after the start of fever, the client noted yellowish discoloration of his sclera. History showed that he waded in flood waters about 2 weeks before the onset of symptoms. Based on her history, which disease condition will you suspect?

a. Hepatitis A b. Hepatitis B c. Tetanus d. Leptospirosis

50.Mickey a 3-year old client was brought to the health center with the chief complaint of severe diarrhea and the passage of rice water stools. The client is most probably suffering from which condition?

a. Giardiasis b. Cholera c. Amebiasis d. Dysentery

51.The most prevalent form of meningitis among children aged 2 months to 3 years is caused by which microorganism?

a. Hemophilus influenzae b. Morbillivirus c. Steptococcus pneumoniae d. Neisseria meningitidis

52.The student nurse is aware that the pathognomonic sign of measles is Kopliks spot and you may see Kopliks spot by inspecting the:

a. Nasal mucosa b. Buccal mucosa c. Skin on the abdomen d. Skin on neck

53.Angel was diagnosed as having Dengue fever. You will say that there is slow capillary refill when the color of the nailbed that you pressed does not return within how many seconds?

a. 3 seconds b. 6 seconds c. 9 seconds d. 10 seconds

54.In Integrated Management of Childhood Illness, the nurse is aware that the severe conditions generally require urgent referral to a hospital. Which of the following severe conditions DOES NOT always require urgent referral to a hospital?

a. Mastoiditis b. Severe dehydration c. Severe pneumonia d. Severe febrile disease

55.Myrna a public health nurse will conduct outreach immunization in a barangay Masay with a population of about 1500. The estimated number of infants in the barangay would be:

a. 45 infants b. 50 infants c. 55 infants d. 65 infants

56.The community nurse is aware that the biological used in Expanded Program on Immunization (EPI) should NOT be stored in the freezer?

a. DPT b. Oral polio vaccine c. Measles vaccine d. MMR

57.It is the most effective way of controlling schistosomiasis in an endemic area?

a. Use of molluscicides b. Building of foot bridges c. Proper use of sanitary toilets d. Use of protective footwear, such as rubber boots

58.Several clients is newly admitted and diagnosed with leprosy. Which of the following clients should be classified as a case of multibacillary leprosy?

a. 3 skin lesions, negative slit skin smear b. 3 skin lesions, positive slit skin smear c. 5 skin lesions, negative slit skin smear d. 5 skin lesions, positive slit skin smear

59.Nurses are aware that diagnosis of leprosy is highly dependent on recognition of symptoms. Which of the following is an early sign of leprosy?

a. Macular lesions b. Inability to close eyelids c. Thickened painful nerves d. Sinking of the nosebridge

60.Marie brought her 10 month old infant for consultation because of fever, started 4 days prior to consultation. In determining malaria risk, what will you do?

a. Perform a tourniquet test. b. Ask where the family resides. c. Get a specimen for blood smear. d. Ask if the fever is present everyday.

61.Susie brought her 4 years old daughter to the RHU because of cough and colds. Following the IMCI assessment guide, which of the following is a danger sign that indicates the need for urgent referral to a hospital?

a. Inability to drink b. High grade fever c. Signs of severe dehydration d. Cough for more than 30 days

62.Jimmy a 2-year old child revealed baggy pants. As a nurse, using the IMCI guidelines, how will you manage Jimmy?

a. Refer the child urgently to a hospital for confinement. b. Coordinate with the social worker to enroll the child in a feeding program. c. Make a teaching plan for the mother, focusing on menu planning for her child.

d. Assess and treat the child for health problems like infections and intestinal parasitism.

63.Gina is using Oresol in the management of diarrhea of her 3-year old child. She asked you what to do if her child vomits. As a nurse you will tell her to:

a. Bring the child to the nearest hospital for further assessment. b. Bring the child to the health center for intravenous fluid therapy. c. Bring the child to the health center for assessment by the physician. d. Let the child rest for 10 minutes then continue giving Oresol more slowly.

64.Nikki a 5-month old infant was brought by his mother to the health center because of diarrhea for 4 to 5 times a day. Her skin goes back slowly after a skin pinch and her eyes are sunken. Using the IMCI guidelines, you will classify this infant in which category?

a. No signs of dehydration b. Some dehydration c. Severe dehydration d. The data is insufficient.

65.Chris a 4-month old infant was brought by her mother to the health center because of cough. His respiratory rate is 42/minute. Using the Integrated Management of Child Illness (IMCI) guidelines of assessment, his breathing is considered as:

a. Fast b. Slow c. Normal d. Insignificant

66.Maylene had just received her 4th dose of tetanus toxoid. She is aware that her baby will have protection against tetanus for

a. 1 year b. 3 years c. 5 years d. Lifetime

67.Nurse Ron is aware that unused BCG should be discarded after how many hours of reconstitution?

a. 2 hours b. 4 hours c. 8 hours d. At the end of the day

68.The nurse explains to a breastfeeding mother that breast milk is sufficient for all of the babys nutrient needs only up to:

a. 5 months b. 6 months c. 1 year d. 2 years

69.Nurse Ron is aware that the gestational age of a conceptus that is considered viable (able to live outside the womb) is:

a. 8 weeks b. 12 weeks c. 24 weeks d. 32 weeks

70.When teaching parents of a neonate the proper position for the neonates sleep, the nurse Patricia stresses the importance of placing the neonate on his back to reduce the risk of which of the following?

a. Aspiration b. Sudden infant death syndrome (SIDS) c. Suffocation d. Gastroesophageal reflux (GER)

71.Which finding might be seen in baby James a neonate suspected of having an infection?

a. Flushed cheeks b. Increased temperature c. Decreased temperature d. Increased activity level

72.Baby Jenny who is small-for-gestation is at increased risk during the transitional period for which complication?

a. Anemia probably due to chronic fetal hyposia b. Hyperthermia due to decreased glycogen stores c. Hyperglycemia due to decreased glycogen stores d. Polycythemia probably due to chronic fetal hypoxia

73.Marjorie has just given birth at 42 weeks gestation. When the nurse assessing the neonate, which physical finding is expected?

a. A sleepy, lethargic baby b. Lanugo covering the body c. Desquamation of the epidermis d. Vernix caseosa covering the body

74.After reviewing the Myrnas maternal history of magnesium sulfate during labor, which condition would nurse Richard anticipate as a potential problem in the neonate?

a. Hypoglycemia b. Jitteriness c. Respiratory depression d. Tachycardia

75.Which symptom would indicate the Baby Alexandra was adapting appropriately to extra-uterine life without difficulty?

a. Nasal flaring b. Light audible grunting c. Respiratory rate 40 to 60 breaths/minute d. Respiratory rate 60 to 80 breaths/minute

76. When teaching umbilical cord care for Jennifer a new mother, the nurse Jenny would include which information?

a. Apply peroxide to the cord with each diaper change b. Cover the cord with petroleum jelly after bathing c. Keep the cord dry and open to air d. Wash the cord with soap and water each day during a tub bath.

77.Nurse John is performing an assessment on a neonate. Which of the following findings is considered common in the healthy neonate?

a. Simian crease b. Conjunctival hemorrhage c. Cystic hygroma d. Bulging fontanelle

78.Dr. Esteves decides to artificially rupture the membranes of a mother who is on labor. Following this procedure, the nurse Hazel checks the fetal heart tones for which the following reasons?

a. To determine fetal well-being. b. To assess for prolapsed cord c. To assess fetal position d. To prepare for an imminent delivery.

79.Which of the following would be least likely to indicate anticipated bonding behaviors by new parents?

a. The parents willingness to touch and hold the new born. b. The parents expression of interest about the size of the new born. c. The parents indication that they want to see the newborn. d. The parents interactions with each other.

80.Following a precipitous delivery, examination of the client's vagina reveals

a fourth-degree laceration. Which of the following would be contraindicated when caring for this client? a. Applying cold to limit edema during the first 12 to 24 hours. b. Instructing the client to use two or more peripads to cushion the area. c. Instructing the client on the use of sitz baths if ordered. d. Instructing the client about the importance of perineal (kegel) exercises.

81. A pregnant woman accompanied by her husband, seeks admission to the labor and delivery area. She states that she's in labor and says she attended the facility clinic for prenatal care. Which question should the nurse Oliver ask her first?

a. Do you have any chronic illnesses? b. Do you have any allergies? c. What is your expected due date? d. Who will be with you during labor?

82.A neonate begins to gag and turns a dusky color. What should the nurse do first?

a. Calm the neonate. b. Notify the physician. c. Provide oxygen via face mask as ordered d. Aspirate the neonates nose and mouth with a bulb syringe.

83. When a client states that her "water broke," which of the following actions would be inappropriate for the nurse to do?

a. Observing the pooling of straw-colored fluid. b. Checking vaginal discharge with nitrazine paper. c. Conducting a bedside ultrasound for an amniotic fluid index. d. Observing for flakes of vernix in the vaginal discharge.

84. A baby girl is born 8 weeks premature. At birth, she has no spontaneous respirations but is successfully resuscitated. Within several hours she develops respiratory grunting, cyanosis, tachypnea, nasal flaring, and retractions. She's diagnosed with respiratory distress syndrome, intubated, and placed on a ventilator. Which nursing action should be included in the baby's plan of care to prevent retinopathy of prematurity?

a. Cover his eyes while receiving oxygen. b. Keep her body temperature low. c. Monitor partial pressure of oxygen (Pao2) levels. d. Humidify the oxygen.

85. Which of the following is normal newborn calorie intake?

a. 110 to 130 calories per kg. b. 30 to 40 calories per lb of body weight. c. At least 2 ml per feeding d. 90 to 100 calories per kg

86. Nurse John is knowledgeable that usually individual twins will grow appropriately and at the same rate as singletons until how many weeks?

a. 16 to 18 weeks b. 18 to 22 weeks c. 30 to 32 weeks d. 38 to 40 weeks

87. Which of the following classifications applies to monozygotic twins for whom the cleavage of the fertilized ovum occurs more than 13 days after fertilization?

a. conjoined twins b. diamniotic dichorionic twins c. diamniotic monochorionic twin d. monoamniotic monochorionic twins

88. Tyra experienced painless vaginal bleeding has just been diagnosed as having a placenta previa. Which of the following procedures is usually performed to diagnose placenta previa?

a. Amniocentesis b. Digital or speculum examination c. External fetal monitoring d. Ultrasound

89. Nurse Arnold knows that the following changes in respiratory functioning during pregnancy is considered normal:

a. Increased tidal volume b. Increased expiratory volume c. Decreased inspiratory capacity d. Decreased oxygen consumption

90. Emily has gestational diabetes and it is usually managed by which of the following therapy?

a. Diet b. Long-acting insulin c. Oral hypoglycemic d. Oral hypoglycemic drug and insulin

91. Magnesium sulfate is given to Jemma with preeclampsia to prevent which of the following condition?

a. Hemorrhage b. Hypertension c. Hypomagnesemia d. Seizure

92. Cammile with sickle cell anemia has an increased risk for having a sickle cell crisis during pregnancy. Aggressive management of a sickle cell crisis includes which of the following measures?

a. Antihypertensive agents b. Diuretic agents c. I.V. fluids d. Acetaminophen (Tylenol) for pain

93. Which of the following drugs is the antidote for magnesium toxicity?

a. Calcium gluconate (Kalcinate) b. Hydralazine (Apresoline) c. Naloxone (Narcan) d. Rho (D) immune globulin (RhoGAM)

94. Marlyn is screened for tuberculosis during her first prenatal visit. An intradermal injection of purified protein derivative (PPD) of the tuberculin bacilli is given. She is considered to have a positive test for which of the following results?

a. An indurated wheal under 10 mm in diameter appears in 6 to 12 hours. b. An indurated wheal over 10 mm in diameter appears in 48 to 72 hours. c. A flat circumcised area under 10 mm in diameter appears in 6 to 12 hours. d. A flat circumcised area over 10 mm in diameter appears in 48 to 72 hours.

95. Dianne, 24 year-old is 27 weeks pregnant arrives at her physicians office with complaints of fever, nausea, vomiting, malaise, unilateral flank pain, and costovertebral angle tenderness. Which of the following diagnoses is most likely?

a. Asymptomatic bacteriuria b. Bacterial vaginosis c. Pyelonephritis d. Urinary tract infection (UTI)

96. Rh isoimmunization in a pregnant client develops during which of the following conditions?

a. Rh-positive maternal blood crosses into fetal blood, stimulating fetal antibodies. b. Rh-positive fetal blood crosses into maternal blood, stimulating maternal antibodies. c. Rh-negative fetal blood crosses into maternal blood, stimulating maternal antibodies. d. Rh-negative maternal blood crosses into fetal blood, stimulating fetal antibodies.

97. To promote comfort during labor, the nurse John advises a client to assume certain positions and avoid others. Which position may cause maternal hypotension and fetal hypoxia?

a. Lateral position b. Squatting position

c. Supine position d. Standing position

98. Celeste who used heroin during her pregnancy delivers a neonate. When assessing the neonate, the nurse Lhynnette expects to find:

a. Lethargy 2 days after birth. b. Irritability and poor sucking. c. A flattened nose, small eyes, and thin lips. d. Congenital defects such as limb anomalies.

99. The uterus returns to the pelvic cavity in which of the following time frames?

a. 7th to 9th day postpartum. b. 2 weeks postpartum. c. End of 6th week postpartum. d. When the lochia changes to alba.

100. Maureen, a primigravida client, age 20, has just completed a difficult, forcepsassisted delivery of twins. Her labor was unusually long and required oxytocin (Pitocin) augmentation. The nurse who's caring for her should stay alert for:

a. Uterine inversion b. Uterine atony c. Uterine involution

d. Uterine discomfort OB 1. A woman of 38 weeks AOG is experiencing true labor when her contraction pattern shows: occasional irregular contractions irregular contractions that increase in intensity regular contractions that remain the same regular contractions that increase in frequency and duration

2.

The nurse should encourage her gravid patient to void frequency during labor, primarily to: prevent urinary infections enhance fetal descent strengthen the vaginal and perineal muscles assess urine specimens for albumin

3.

The placenta forms from the: chorionic villi and deciduas vera chorionic villi and decidua capsularis deciduas basalis and deciduas vera chorionic villi and decidua basalis

4.

When performing Leopold's maneuvers, which of the following would the nurse ask the client to do to ensure optimal comfort and accuracy? breathe deeply for one minute

empty her bladder drink a full glass of water lie on her left side

5.

The nurse instructs a primigravid patient to increase her intake of Magnesium because of its role with which of the following? prevention of demineralization of the mother's bones synthesis of proteins and nucleic acids and fats amino acid metabolism synthesis of neural pathways in the fetus

6.

After instructing a primigravid client about the functions of the placenta, the nurse determines that the client needs additional teaching when she says that which of the following hormones is produced by the placenta: testosterone estrogen progesterone HcG

7.

During a childbirth preparation class, a primigravid client at 36 weeks gestation tells the nurse, "My lower back has really been bothering me lately." Which of the following exercises suggested by the nurse would be most helpful? pelvic rocking deep breathing tailor sitting squatting

8.

What is a common endocrine response to pregnancy? decrease cortisol levels decrease production of prolactin increase plasma parathyroid hormone increase maternal blood glucose level

9.

Combined oral contraceptives prevent pregnancy by inhibiting the production of: FSH & prolactin LH & estrogen FSH & LH Estrogen and progesterone

10. The nurse should instruct her client to discontinue the oral contraceptive and call the physician immediately if she experiences: hypomenorrhea dysmenorrhea severe headache leucorrhea

11. Assessment of primigravid client reveals cervical dilatation at 8cm and complete effacement. The client complains of severe back pain during this phase of labor. The nurse explains that the client's severe back pain is most likely caused by the fetal occiput being in a position that is identified as which of the following? breech

transverse posterior anterior

12. While the nurse is caring for a multiparous client in active labor at 37 weeks gestation, the client tells the nurse, "I think my water just broke." Which of the following would the nurse do first? turn the client on the right side assess the fetal heart rate pattern assess the color, amount and odor of fluid check the client's cervical dilation.

13. The physician orders oxytocin to be added to the IVF of a 26-year old multigravid client at 38 weeks AOG after vaginal delivery. The nurse anticipates administering the oxytocin after delivery of which of the following first placenta second placenta first twin second twin

14. While making a home visit to a postpartum client on day 10, the nurse would anticipate that the client's lochia would be: dark red pink brown white

15. Which of the following forms the basis for the teaching plan about avoiding medication use unless prescribed for a primiparous client who is breastfeeding? breast milk quality and richness are decreased the mother's motivation to breastfeed is diminished medications may be excreted in breastmilk to the nursing neonate medication interfere with the mother's let-down reflex

16. A client is admitted to the hospital with contractions that are about 1-2 minutes apart and last for 60 seconds. Vaginal exam reveals that her cervix is dilated 8cm. The client is in which stage labor? latent phase active phase third stage transitional phase

17. During the third postpartum day, which of the following would the nurse be most likely to find in the client: she's interested in learning more about newborn care she talks a lot about the birth experience she sleeps whenever the baby isn't present she requests help in choosing a name for the baby

18. When assessing a client's episiotomy, the nurse should be especially careful to observe: location

discharge and odor edema and approximation subinvolution

19. In performing a routine fundal assessment, the nurse finds that the client's fundus is boggy. The nurse should first: call the physician massage the fundus assess the lochial flow obtain an order for methelergonovine

20. Which assessment of a woman in labor can be determined by vaginal examination? fetal weight cervical dilatation strength of contraction fetal head circumference

21. The postovulation rise in BBT is due to the high blood level of which hormone? FSH HPL estrogen progesterone

22. A pregnant woman's history reveals one pregnancy, terminated by elective abortion at 10 weeks, birth of twins at 37 weeks and a spontaneous abortion at

12 weeks. According to TPAL system, which of the following describes her present parity. 0-2-2-2 2-0-2-2 0-1-2-2 1-0-2-2

23. Which principal factor cause vaginal patient to be acidic? cervical mucus changes secretion from skene's glands the action of doderlein's bacillus secretions from Bartholin's gland

24. Which normal assessment finding can the nurse expect in the 34th week of pregnancy? Braxton-Hicks contractions, joint hypermobility and backache Dysuria, constipation, hemorrhoids and lightening Feeling of tranquility and heightened introspection Morning sickness, breast tenderness

25. What are the 2 fetal membranes? ectoderm and mesoderm chorion and amnion chorion and endoderm amnion and chorionic villi

Chn and Ob 3 Situation 1: Mariah is a 31 year old lawyer who has been married for 6 moths. She consults you for guidance in relation with her menstrual cycle and her desire to get pregnant. 1. She wants to know the length of her menstrual cycle. Her periodic menstrual period is October 22 to 26. Her LMB is November 21. Which of the following number of days will be your correct response? A. 29 B. 28 C. 30 D. 31 2. You advised her to observe and record the signs of Ovulation. Which of the following signs will she likely note down? 1. A 1 degree Farenheit rise in basal body temperature 2. Cervical mucus becomes copious and clear 3. One pound increase in weight 4. Mitteischmerz A. 1,2,4 B. 1,2,3 C. 2,3,4 D. 1,3,4 3. You instruct Mariah to keep record of her basal temperature everyday, which of the following instructions is incorrect? A. If coitus has occured, this should be reflected in the chart B. It is best to have coitus on the evening following a drop in BBT to become pregnant

C. Temperature should be taken immediately after waking and before getting put of bed D. BBT is lowest during the secretory phase 4. She reports an increase in BBT on December 16. Which hormone brings about this change in her BBT? A. Estrogen B. Progesterone C. Gonadootrophine D. Follicle Stimulating Hormone 5. The following month, Mariah suspects she is pregnant. Her urine is positive for Human Chorionic Gonadotrophin. Which structure produce HCG? A. Pituitary Gland B. Trophoblastic cells of the embryo C. Uterine deciduas D. Ovarian follicles Situation 2: Mariah came back and she is now pregnant. 6. At 5 moths gestation, which of the following fetal development would probably be expected: A. Fetal development are felt by Mariah B. Vernix caseosa covers the entire body C. Viable if delivered within this period D. Braxton hicks contractions are observed 7. The nurse palpates the abdomen of Mariah. Now at 5 month gestation, what level of the abdomen can be the fundic height be palpated? A. Symphysis pubis B. Midpoint between the umbilicus and the xiphoid process C. Midpoint between the Symphysis pubis the umbilicus D. Umbilicus

8. She worries about her small breast, thinking that she probably will incapable to breastfeed her baby. Which of the following responses of the nurse is correct? A. The size of your breast will not affect your lactation. B. You can switch to bottle feeding. C. You can try to have exercise to increase the size of your breast. D. Manual expression of milk is possible. 9. She tells the nurse that she does not take milk regularly. She claims that she does not want to gain too much weight during her pregnancy. Which of the following nursing diagnosis is a priority? A. Potential self-esteem disturbance related to physiologic changes in pregnancy B. Ineffective individual coping related to physiologic changes in pregnancy C. Fear related to the effects of pregnancy D. Knowledge deficit regarding nutritional requirements pregnancies related to lack of information sources. 10. Which of the following interventions will likely ensure compliance of Mariah? A. Incorporate her food preferences that are adequately nutritious in her meal plan. B. Consistently counsel toward optimum nutritional intake C. Respect her right to reject dietary information if she chooses D. Information of the adverse effects of inadequate nutrition to her fetus Situation 3: Susan is a patient in the clinic where you work. She is inquiring about pregnancy. 11. Susan tells you she is worried because she develops breast later than most of her friends. Breast development is termed as: A. Adrenarche B. Mamarche C. Thelarche D. Menarche 12. Kevin, Susans husband tells you that he is considering vasectomy. After the birth of their new child. Vasectomy involves the incision of which organ?

A. The testes B. The vas deferens C. The epididymis D. The scrotum 13. On examination, Susan has been found of having a cystocele. A cystocele is: A. A sebaceous cyst arising from the vulvar fold. B. Protrusion of intestines into the vagina C. Prolapse of the uterus into the vagina D. Herniation of the bladder into the vaginal wall. 14. Susan typically has menstrual cycle of 34 days. She told you she had a coitus on days 8, 10 and 20 of her menstrual cycle. Which is the day on which she is most likely to conceive? A. 8th day B. 10th day C. Day 15 D. Day 20 15. While taking with Susan, 2 new patients arrived and they are covered with large towels and the nurse noticed that there are many cameraman and news people outside of the OPD. Upon assessment the nurse noticed that both of them are still nude and the male clients penis is still inside the female clients vagina and the male client said that I cant pull it. Vaginismus was your first impression. You know that the psychological cause of Vaginismus is related to: A. The male client inserted the penis too deeply that it stimulates vaginal closure B. The penis was too large thats why the vagina triggered its defense to attempt to close C. The vagina do not want to be penetrated D. It is due to learning patterns of the female client where she views sex as bad or sinful. Situation 4: Overpopulation is one problem in the Philippines that case economic drain. Most Filipinos are against in legalizing abortion. As a nurse, Mastery of contraception is needed to contribute to the society and economic growth.

16. Supposed that Dana, 17 years old, tells you she wants to use fertility awareness method of contraception. How will she determine her fertile days? A. She will notice that she feels hot as if she has an elevated temperature B. She should assess whether her cervical mucus is thin colour, clear and watery. C. She should monitor her emotions fro sudden anger or crying D. She should assess whether her breast feel sensitive to cool air. 17. Dana chooses to use COC as her family planning method, what is the danger sign of COC you would ask her to report? A. A stuffy or runny nose B. Arthritis like symptoms C. Slight weight gain D. Migraine headache 18. Dana asks about subcutaneous implants and she asks how long will these implants be effective. Your best answer is: A. One month B. Twelve month C. Five years D. 10 years 19. Dana asks about female condoms. Which of the following is true with regards to female condoms? A. The hormone the condom releases might cause mild weight gain. B. She should insert the condom before any penile penetration C. She should coat the condom with spermecide before use D. Female condoms unlike male condoms are reusable. 20. Dana has asked about GIFT procedure. What makes her a good candidate for GIFT? A. She has patent fallopian tubes, so fertilized ova can be implanted on them. B. She is RH negative, a necessary stipulation to rule out RH incompatibity.

C. She has normal uterus, so the sperm can be injected through the cervix into it. D. Her husband is taking sildenafil, so all sperms will be motile. Situation 5 Nurse Lorena is a Family Planning and Infertility Nurse Specialist and currently attends to FAMILY PLANNING CLIENTS AND INFERTILE COUPLES. The following conditions pertain to meeting the nursing needs of this particular population group. 21. Dina, 17 years old, asks you how a tubal ligation prevents pregnancy. Which would be the best answer? A. Prostaglandins released from the cut fallopian tubes can kill sperm B. Sperm can not enter the uterus because the cervical entrance is blocked. C. Sperm can no longer reach the ova, because the fallopian tubes are blocked D. The ovary no longer releases ova as there is no where for them to go. 22. The Dators are a couple undergoing testing for infertility. Infertility is said to exist when: A. a woman has no uterus B. a woman has no children C. a couple has been trying to conceive for 1 year D. a couple has wanted a child for 6 months 23. Another client named Lilia is diagnosed as having endometriosis. This condition interferes with fertility because: A. endometrial implants can block the fallopian tubes B. the uterine cervix becomes inflamed and swollen C. the ovaries stop producing adequate estrogen D. pressure on the pituitary leads to decreased FSH levels 24. Lilia is scheduled to have a hysterosalphingogram. Which of the following instructions would you give her regarding this procedure? A. She will not be able to conceive for 3 months after the procedure B. The sonogram of the uterus will reveal any tumors present C. Many women experience mild bleeding as an after effect

D. She may feel some cramping when the dye is inserted 25. Lilias cousin on the other hand, knowing nurse Lorenas specialization asks what artificial insemination by donor entails. Which would be your best answer if you were Nurse Lorena? A. Donor sperm are introduced vaginally into the uterus or cervix B. Donor sperm are injected intra-abdominally into each ovary C. Artificial sperm are injected vaginally to test tubal patency D. The husbands sperm is administered intravenously weekly Situation 6: You are assigned to take care of a group of patients across the lifespan. 26. Pain in the elder persons require careful assessment because they: A. Experienced reduce sensory perception B. Have increased sensory perception C. Are expected to experience chronic pain D. Have decreased pain threshold 27. Administration of analgesics to the older persons require careful patient assessment because older people: A. Are more sensitive drugs B. Have increased hepatic, renal, and gastrointestinal function C. Have increased sensory perception D. Mobilize drugs more rapidly 28. The elderly patient is at higher risk for urinary incontinence because: A. Increased glomerular filtration B. Diuretic use C. Decreased bladder capacity 29. Which of the following is the MOST COMMON sign of infection among the elderly? A. Decreased breath sounds with crackles B. Fever

C. Pain D. Change in the mental status 30. Priorities when caring for the elderly trauma patient: A. Circulation, airway, breathing B. Disability(Neurologic), airway, breathing C. Airway, Breathing, Disability(Neurologic), D. Airway, breathing, Circulation 31. Preschoolers are able to see things from which of the following perspectives? A. Their peers B. Their own and their caregivers C. Their own and their mothers D. Only their own 32. In conflict management, the win-win approach occurs when: A. There are two conflicts and the parties agree to each one B. Each party gives in on 50% of the disagreement making the conflict C. Both parties involved are committed in solving the conflict D. The conflict is settled out of court so the legal system mandates parties win. 33. According to the social-International perspective of child abuse and neglect, four factors place the family members at risk for abuse, these risk factors are the family members at risk for abuse. These risk factors are the family itself, the caregiver, the child and: A. The presence of a family crisis B. Genetics C. The national emphasis on sex D. Chronic poverty 34. Which of the following signs and symptoms would you most likely find when assessing an infant with Arnold-Chari malformation? A. Weakness of the leg muscles, loss of sensation in the legs, and restlessness

B. Difficulty swallowing, diminished or absent gag reflex and respiratory distress C. Difficulty sleeping, hypervigilant and an arching of the back D. Paradoxical irritability, diarrhea and vomiting 35. A parent calls you and frantically reports that her child has gotten into her famous ferrous sulfate pills and ingested a number of these pills. Her child is now vomiting, has bloody diarrhea and is complaining of abdominal pain. You will tell the mother to: A. Call emergency medical services (EMS) and get the child to the emergency room B. Relax because these symptoms will pass and the child will be fine C. Administer syrup of pecac D. Call the poison control center 36. A client says she heard from a friend that you stop having periods once you are on the pill. The most appropriate response would be: A. The pill prevents the uterus from making such endometrial lining, that is why period may often be scant or skipped occasionally. B. If your friend has missed her period, she should stop taking the pills and get a pregnancy test. C. The pill should cause a normal menstrual period every month. It sounds like your friend has not been taking the pills properly. D. Missed period can be very dangerous and may lead to the formation of precancerous cells. 37. The nurse assessing newborn babies and infants during their hospital stay notice which of the following symptoms as a primary manifestation of Hirschprungs disorder? A. A fine rash over the trunk B. Failure to pass meconium during the first 24 hours after birth C. The skin turns yellow and then brown over the first 24 hours to 46 hours after birth. D. High grade fever.

38. A client is 7 months pregnant and has just been diagnosed as having a partial placenta previa. She is able and has minimal spotting and is being sent home. Which of these instructions to the client may indicate a need for further teaching? A. Maintain bed rest with bathroom privileges B. Avoid intercourse for three days C. Call if contractions occur. D. Stay on left side as much as possible when lying down. 39. A woman has been rushed to the hospital with ruptured membrane. Which of the following should the nurse check first? A. Check for the presence of infarction. B. Assess for Prolapse of the umbilical cord C. Check the maternal heart rate D. Assess the color of the amniotic fluid 40. The nurse notes that the infant is wearing a plastic-coated diaper. If a topical medication were to be prescribed and it were to go on the stomachs or buttocks, the nurse would teach the caregivers to: A. Avoid covering the area of the topical medication with the diaper B. Avoid the use of clothing on top of the diaper C. Put the diaper on as usual D. Apply an icepack for 5 minutes to the outside of the diaper 41. Which of the following factors is most important in determining the success of relationships used in delivering nursing care? A. Type of illness of the client B. Transference and counter Transference C. Effective communication D. Personality of the participants 42. Grace sustained a laceration on her leg from automobile accident. Why are lacerations of lower extremities potentially more serious among pregnant women than other?

A. lacerations can provoke allergic responses due to gonadotropic hormone release B. a woman is less able to keep the laceration clean because of her fatigue C. healing is limited during pregnancy so these will not heal until after birth D. increased bleeding can occur from uterine pressure on leg veins 43. In working with the caregivers of a client with an cute or chronic illness, the nurse would: A. Teach care daily and let the caregivers do a return demonstration just before discharge B. Difficulty swallowing, diminished or absent gag reflex and respiratory distress C. Difficulty sleeping, hypervigilant and arching of the back D. Paradoxical irritability, diarrhea and vomiting 44. Which of the following roles BEST exemplifies the expanded role of the nurse? A. Circulating nurse in surgery B. Medication nurse C. Obstretical nurse D. Pediatric nurse practitioner 45. According to De Rosa and Kochuras (2006) article entitled Implement Culturally Health Care in your workplace, cultures have different patterns of verbal and nonverbal communication. Which difference does NOT necessarily belong? A. Personal behaviour B. Eye contact C. Subject Matter D. Conversational style 46. You are the nurse assigned to work with a child with acute glomerulonephritis. By following the prescribed treatment regimen, the child experiences a remission. You are now checking to make sure the child does not have a relapse. Which finding most lead you to the conclusion that a relapse is happening? A. Elevated temperature, cough, sore throat, changing complete blood count (CBC) with differential count

B. A urine dipstick measurement of 2+ proteinuria or more for 3 days or the child found to have 3-4+ proteinuria plus edema. C. The urine dipstick showing glucose in the urine for 3 days, extreme thirst, increase in urine output and a moon face. D. A temperature of 37.8 degrees (100 degrees F) flank pain, burning frequency, urgency on voiding and cloudy urine. 47. The nurse is working with an adolescent who complains of being lonely and having a lack of fulfilment in her life. This adolescent shies away from intimate relationships at times yet at other times she appears promiscuous. The nurse will likely work with this adolescent in which of the areas? A. Isolation B. Loneliness C. Lack of fulfilment D. Identity 48. The use of interpersonal decision making psychomotor skills and application of knowledge expected in the role of a licensed health care professional in the context of public health welfare and safety as an example of? A. Delegation B. Supervision C. Responsibility D. Competence 49. The painful phenomenon known as back labor occurs in a client whose fetus in what position? A. Brow position B. Right occipito-Anterior Position C. Breech position D. Left occipito-Posterior Position 50. Focus methodology stands for? A. Focus, Organize, Clarify, Understand and Solution B. Focus, Opportunity, Continuous, Utilize, Substantiate

C. Focus, Organize, Clarify, Understand, Substantiate D. Focus, Opportunity, Continuous (process), Understand, Solution Situation 7: The infant and child mortality rate in the low to middle income countries is ten times higher than industrialized countries. In response to this the WHO and UNICEF launched protocol Integrated Management of Childhood Illness to reduce the morbidity and mortality against childhood illnesses. 51. If a child with diarrhea registers two signs in the yellow row in the IMCCI char, we can classify the patient as: A. Moderate dehydration B. Some dehydration C. Severe dehydration D. No dehydration 52. Celeste has had diarrhea for 8 days. There is no blood in the stool, he is irritable, his eyes are sunken, the nurse offers fluid to Celeste and he drinks eagerly. When the nurse pinched the abdomen it goes back slowly. How will you classify Celestes Illness? A. Moderate dehydration B. Some dehydration C. Severe dehydration D. No dehydration 53. A child who is 7 weeks has had diarrhea for 14 days but has no sign of dehydration is classified as? A. Persistent diarrhea B. Severe dysentery C. Dysentery D. Severe Persistent diarrhea 54. The child with no dehydration needs home treatment. Which of the following is not included in the rules for home treatment in this case? A. Forced fluids B. When to return

C. Give Vitamin A supplement D. Feeding more 55. Fever as used in IMCI includes: A. Axillary temperature of 37.5 or higher B. Rectal temperature of 38 or higher C. Feeling hot to touch D. All of the above E. A and C only Situation: Prevention of Dengue is an important nursing responsibility and controlling its spread is priority once outbreak has been observed. 56. An important role of the community health nurse in the prevention and control of Dengue H-fever includes: A. Advising the elimination of vectors by keeping water containers covered B. Conducting strong health education drives/campaign directed toward proper garbage disposal C. Explaining to the individuals, families, groups and community the nature of the disease and its causation. D. Practicing residual spraying with insectesides 57. Community health nurses should be alert in observing a Dengue suspect. The following is NOT an indicator for hospitalization of H-fever suspects? A. Marked anorexia, abdominal pain and vomiting B. Increasing hematocrit count C. Cough of 30 days D. Persistent headache 58. The community health nurses primary concern in the immediate control of hemmorrhage among patients with dengue is: A. Advising low fiber and non-fat diet B. Providing warmth through light weight covers C. Observing closely the patient for vital signs leading to shock

D. Keeping the patient at rest. 59. Which of these signs may NOT be REGARDED as a truly positive signs indicative of Dengue H-fever? A. Prolonged Bleeding Time B. Appearance of at least 20 petechiae within 1 cm square C. Steadily increasing hematocrit count D. Fall in the platelet count 60. Which of the following is the most important treatment of patients with Dengue H-fever? A. Give aspirin for fever B. Replacement of body fluids C. Avoid unnecessary movement D. Ice cap over abdomen in case of melena Situation 9: Health education and Health Promotion is an important part of nursing responsibility in the community. Immunization is a form of health promotion that aims at preventingthe common childhood illnesses. 61. In correcting misconception and myths about certain diseases and their management, the health worker should first: A. Identify the myths and misconceptions prevailing in the community B. Identify the source of these myths and misconceptions C. Explain how and why these myths came about D. Select the appropriate IEC strategies to correct them. 62. How many percent of measles are prevented by immunization at 9 months age? A. 80 % B. 90% C. 99 % D. 95 % 63. After TT3 vaccination a mother is said to be protected to tetanus by around?

A. 80 % B. 85 % C. 99 % D. 90 % 64. If ever convulsion occurs after administering DPT, what should nurse best suggest to the mother? A. Do not continue DPT vaccination anymore B. Advise mother to come back aster 1 week C. Give DT instead of DPT D. Give pertussis of the DPT and remove DT 65. These vaccines are given 3 doses at one month intervals: A. DPT, BCG, TT B. DPT, TT, OPV C. OPV, Hep. B, DPT D. Measles, OPV, DPT Situation 10: With the increasing documented cases of CANCER the best alternative to treatment still remains to be PREVENTION. The following conditions apply. 66. Which among the following is the primary focus of prevention of cancer? A. Elimination of conditions causing cancer B. Diagnosis and treatment C. Treatment at early stage D. Early detection 67. In the prevention and control of cancer, which of the following activity is the most important function of the community health nurse? A. Conduct community assemblies B. Referral to cancer specialist those clients with symptoms of cancer C. Use the nine warning signs of cancer as parameters in our process of detection; control and treatment modalities.

D. Teach woman about proper/correct nutrition. 68. Who among the following are recipients of the secondary level of care for cancer cases? A. Those under early case detection B. Those under supportive care C. Those scheduled for surgery D. Those under going treatment 69. Who among the following are recipients of the tertiary level of care for cancer cases? A. Those under early treatment B. Those under supportive care C. Those under early detection D. Those scheduled for surgery 70. In Community Health Nursing, despite the availability and use of many equipment and devices to facilitate the job of the community health nurse, the nurse should be prepared to apply is a scientific approach. This approach ensures quality of care even at the community setting. This nursing parlance is nothing less than the: A. Nursing diagnosis B. Nursing protocol C. Nursing research D. Nursing process Situation 11 Two children were brought to you. One with chest indrawing and the other had diarrhea. The following questions apply: 71. Using Integrated Management and Childhood Illness (IMCI) approach, how would you classify the 1st child? A. Brochopneumonia B. No pneumonia: cough or cold C. Severe pneumonia

D. Pneumonia 72. The 1st child who is 13 months has fast breathing using IMCI parameters he has: A. 40 breaths per minute or more B. 50 breaths per minute C. 30 breaths per minute or more D. 60 breaths per minute 73. Nina, the 2nd child has diarrhea for 5 days. There is no blood in the stool. She is irritable and her eyes are sunken. The nurse offered fluids and the child drinks eagerly. How would you classify Ninas illness? A. Some dehydration B. Dysentery C. Severe dehydration D. No dehydration 74. Ninas treatment should include the following EXCEPT: A. Reassess the child and classify him for dehydration B. For infants under 6 months old who are not breastfed, give 100-200 ml clean water as well during this period. C. Give in the health center the recommended amount of ORS for 4 hours. D. Do not give any other foods to the child for home treatment 75. While on treatment, Nina 18 months old weighed 18 kgs and her temperature registered at 37 degrees C. Her mother says she developed cough 3 days ago. Nina has no general danger signs. She has 45 breaths/minute, no chest indrawing, no stridor. How would you classify Ninas manifestation. A. No pneumonia B. Severe pneumonia C. Pneumonia D. Bronchopneumonia

76. Carol is 15 months old and weighs 5.5 kgs and it is her initial visit. Her mother says that Carol is not eating well and unable to breastfeed, he has no vomiting, has no convulsion and not abnormally sleepy or difficult to awaken. Her temperature is 38.9 deg C. Using the integrated management of childhood illness or IMCI strategy, if you were the nurse in charge of Carol, how will you classify her illness? A. a child at a general danger sign B. very severe febrile disease C. severe pneumonia D. severe malnutrition 77. Why are small for gestational age newborns at risk for difficulty maintaining body temperature? A. their skin is more susceptible to conduction of cold B. they are preterm so are born relatively small in size C. they do not have as many fat stored as other infants D. they are more active than usual so they throw off comes 78. Oxytocin is administered to Rita to augment labor. What are the first symptoms of water intoxication to observe for during this procedure? A. headache and vomiting B. a swollen tender tongue C. a high choking voice D. abdominal bleeding and pain 79. Which of the following treatment should NOT be considered if the child has severe dengue hemorrhagic fever? A. use plan C if there is bleeding from the nose or gums B. give ORS if there is skin Petechiae, persistent vomiting, and positive tourniquet test C. give aspirin D. prevent low blood sugar 80. In assessing the patients condition using the Integrated Management of Childhood Illness approach strategy, the first thing that a nurse should do is to:

A. ask what are the childs problem B. check the patients level of consciousness C. check for the four main symptoms D. check for the general danger signs 81. A child with diarrhea is observed for the following EXCEPT: A. how long the child has diarrhea B. skin Petechiae C. presence of blood in the stool D. signs of dehydration 82. The child with no dehydration needs home treatment. Which of the following is NOT included in the care for home management at this case? A. give drugs every 4 hours B. continue feeding the child C. give the child more fluids D. inform when to return to the health center 83. Ms. Jordan, RN, believes that a patient should be treated as individual. This ethical principle that the patient referred to: A. beneficence B. nonmaleficence C. respect for person D. autonomy 84. When patients cannot make decisions for themselves, the nurse advocate relies on the ethical principle of: A. justice and beneficence B. fidelity and nonmaleficence C. beneficence and nonmaleficence D. fidelity and justice

85. Being a community health nurse, you have the responsibility of participating in protecting the health of people. Consider this situation: Vendors selling bread with their bare hands. They receive money with these hands. You do not see them washing their hands. What should you say/do? A. Miss, may I get the bread myself because you have not washed your hands B. All of these C. Miss, it is better to use a pick up forceps/ bread tong D. Miss, your hands are dirty. Wash your hands first before getting the bread Situation 12: The following questions refer to common clinical encounters experienced by an entry level nurse. 86. A female client asks the nurse about the use of cervical cap. Which statement is correct regarding the use of the cervical cap? A. It may affect Pap smear results B. It does not need to be fitted by the physician C. It does not require the use of spermicide D. It must be removed within 24 hours 87. The major components of the communication process are? A. Verbal, written, and nonverbal B. Speaker, Listener and reply C. Facial expression, tone of voice and gestures D. Message, sender, channel, Receiver and Feedback 88. The extent of burns in children are normally assessed and expressed in terms of: A. The amount of body surface that is unburned B. Percentages of total body surface area (TBSA) C. How deep the deepest burns are D. The severity of the burns on a 1 to 5 burn scale 89. The school nurse notices a child who is wearing old, dirty, poor-fitting clothes; is always hungry; has no lunch money; and is always tired. When the nurse asks the

boy his tiredness, he talks of playing outside until midnight. The nurse will suspect that this child is: A. Being raised by a parent of low intelligence (IQ) B. An orphan C. A victim of child neglect D. The victim of poverty 90. Which of the following indicates the type(s) of acute renal failure? A. Four types: hemorrhagic with and without clotting, and non hemorrhagic with and without clotting B. One type: Acute C. Three types: Prerenal, intrarenal, postrenal D. Two types: Acute and subacute Situation 13: Mike 16 y/o has been diagnose to have AIDS, he worked as entertainer in a cruise ship: 91. Which method of transmission is common to contract AIDS: A. Syringe and needles B. Body fluids C. Sexual contact D. Transfusion 92. Causative organism in AIDS is one of the following: A. Fungus B. Bacteria C. Retrovirus D. Parasites 93. You are assigned in a private room of Mike. Which procedure should be of outmost importance: A. Alcohol wash B. Universal precaution

C. Washing isolation D. Gloving technique 94. What primary health teaching would you give to Mike? A. Daily exercise B. Prevent infection C. Reversal Isolation D. Proper nutrition 95. Exercise precaution must be taken to protect health worker dealing with the AIDS patients, which among these must be done as priority? A. Boil used syringed and needles B. Use gloves when handling specimen C. Label personal belonging D. Avoid accidental wound Situation 14: Michelle is a 6 year old preschooler. She was reported by her sister to have measles but she was at home because of fever, upper respiratory problem and white sports in her mouth. 96. Rubeola is an Arabic term meaning Red, the rash appears on the skin in invasive stage prior to eruption. As a nurse, your physical examination must determine complication especially: A. Otitis media B. Bronchial pneumonia C. Inflammatory conjunctiva D. Membranous laryngitis 97. To render comfort measure is one of the priorities, which includes care of the skin, eyes, ears, mouth and nose. To clean the mouth, your antiseptic is in some form of which one below? A. Water B. Sulfur C. Alkaline

D. Salt 98. As a public health nurse, you teach mother and family members the prevention of complication of measles. Which of the following should be closely watched? A. Temperature fails to drop B. Inflammation of the conjunctiva C. Inflammation of the nasopharynx D. Ulcerative stomatitis 99. Source of infection of measles is secretion of nose and throat of infection person. Filterable of measles is transmitted by: A. Water supply B. Droplet C. Food ingestion D. Sexual contact 100. Method of prevention is to avoid exposure to an infection person. Nursing responsibility for rehabilitation of patient includes the provision of: A. Terminal disinfection B. Injection of gamma globulin C. Immunization D. Comfort measures

CHN OB 4
Situation: Julia, primagravida is brought to the labor room with the following PE findings: Cervical dilation 8cm, fully effaced +1, AOG: 39-40 weeks. 1. When is the first stage of labor considered to be achieved? a) presenting part is at station +1 b) cervix is 10 cm dilated c) uterine contractions occur every 2-3 min. interval d) cervix is gully effaced 2. Upon admission to the labor room, which of the following is not a routine procedure considering

her cervical dilation a) mini prep. of the perineal area b) keep on NPO c) monitor vital signs and FHT d) cleansing enema 3. Which of the following observation requires the nurse to refer stat to the obstetricians? a) frequent urination b) blood-streak mucus in the vaginal discharge c) sudden gush of amniotic fluid from the vagina d) FHT is 110 during uterine contractions but returns to 130 after 10 seconds following contract 4. Which of the following signs indicate that delivery is near? 1. Julie verbalized her desire to defecate 2. uterine contractions increased in frequency duration and intensity 3. the perineum is bulging 4. bloody show is increased a) 1,2,3,4 b) 1,2,3 c) 1,2,4 5. After the delivery of the baby, which of the following indicate placental separation? 1. protrusion of three or more inches of the umbilical cord 2. gradual descent of the uterus further into the pelvis 3. uterus becomes more firm and rounded 4. sudden spurt of blood from the vagina a) 1,3,4 b) 1,2,4 c) 2,3,4 d) 1,2,3 6. The client is in active phase of labor. The physician has administered epidural anesthesia. Which of the following nursing actions should be given highest priority by the nurse? a) ensuring patent IV access line b) checking for rupture of membrane c) monitoring duration of each contraction d) providing food and fluids 7. The client is on her twelfth-week of pregnancy. She had been diagnosed to have ruptured ectopic pregnancy. Which of the following signs and symptoms are characteristic of the condition?

a) profuse bright red vaginal bleeding b) spotting, abdominal pain that radiates to the shoulder c) elevated hemoglobin and hematocrit level d) leukopenia, decreased body temperature 8.You are assessing a 32-week pregnant woman. Which of the following is a biophysical nursing diagnosis? a) body image disturbance b) knowledge deficit c) ambivalence d) alteration in nutrition 9. A nurse provides instructions to a malnourished client regarding iron supplementation during pregnancy. Which statement when made by the client would indicate an understanding of the instructions? a) the iron is best taken on empty stomach b) meat does not provide iron and should be avoided c) iron supplements will give me diarrhea d) my body has all iron it needs and I don't need to take supplement 10. Which of the following meals is best for pregnant woman? a) turkey with green salad b) angel food cake c) hamburger with coffee d) french fries with soda drink 11. Which of the following findings by the nurse indicates that Methergine injection to a client, who had delivered 3 hours ago, is effective? a) uterus is 2 fingerbreadths below the umbilicus b) uterus is boggy and at the level of the umbilicus c) uterus is palpated on the right side of the abdomen d) uterus is 2 fingerbreadths above the umbilicus 12. Which of the following statements when made by the premenopausal woman indicates that she understands the health teachings on breast self-examination (BSE)? a) I will perform breast self-examination every first day of the month b) I will perform breast self-examination 5 days after menstruation c) I will perform breast self-examination 2 to 3 days after the cycle d) I will perform breast self-examination during menstrual period 13. Which of the following signs and symptoms indicates amniotic fluid embolism?

a) sudden onset of respiratory distress, chest pain, BP 90/60 mmHg, RR 26/min, PR 98bpm b) restlessness, chest pain, BP 140/80 mmHg, RR 24/min, PR 70 bpm c) difficulty of breathing, cold, clammy skin, BP 90/60 mmHg, RR 12/min, PR 70 bpm d) chest pain, shortness of breath, BP 160/90 mmHg, RR 28/min, PR 120 bpm 14. The client is in active labor, cervix is 8 cm dilated, is nauseated, and irritable. Which of the following is appropriate nursing action? a) encourage the client to do pant-blow breathing exercises b) encourage the client to push with each contractions c) encourage the client to walk d) encourage the client to turn to the right side 15. The client is in active labor. She is on oxytocin per IV infusion drip. Which of the following situations would require that the infusion be stopped? a) the cervix is 8 cm, dilated, contractions occur every 3-5 minutes b) contractions occur at less than 2 minute intervals or last for longer than 90 seconds c) the cervix is 6 cm, dilated, partially effaced, duration of contractions is 50 to 60 seconds d) contractions occur every 3-5 minutes, last 50 to 60 seconds 16. A pregnant woman is being given magnesium sulfate per slow IV drip. This medication is intended to control a) embolism b) seizures c) bleeding d) uterine contractions 17. Abnormal fetal lie and position were noted. Which of the following procedures does the nurse expect to be arranged first, before external rotation (version) is done? a) amniocentesis b) ultrasound c) fetal heart rate monitoring d) x-ray 18. A client is 32 weeks pregnant. She experiences cord prolapse and is in active labor. Which of the following should be the most immediate action by the nurse? a) push the cord back into the uterus with a gloved hand b) cover cord with sterile dry gauze c) place the client in knee-chest position d) prepare the client for immediate vaginal delivery 19. Who among these pregnant clients is at risk for bleeding?

a) the client who has history of preterm delivery b) the client who has twins in her present pregnancy c) the client who is 18 years of age and is pregnant for the first time d) the client who is pregnant for the third time 20. Which of the following health teachings should be included for a mother who complains of soreness of nipples because of breastfeeding her infant? a) wash your breast with soap and water b) stop breatfeeding for few days c) apply lanolin cream on the nipple d) avoid wearing bra until soreness of nipples disappears 21. Which of the following indicates that Brethine (Theophylline) is effective in a woman on premature labor? a) uterine contractions become more frequent b) uterine contractions stop c) cervical dilatation progresses d) rupture of membrane occurs 22. The woman isi n active labor. The presentation of the fetus is left occiput posterior (LOP). Which of the following measures should be included when caring for the client? a) provide foods and fluids b) assist the client to ambulate c) provide back massage d) allow the client to sleep 23. The postpartum client is bleeding heavily 2 hours after delivery. The fundus of the uterus is firm; uterus at the center of the abdomen. Which of the following actions should the nurse do next? a) change perineal pads b) notify the physician c) massage the uterus d) check perineum 24. The client is on her second trimester of pregnancy. Her BP is 159/95 mmHg. Which of the following would give clue to make a diagnosis? a) weight loss b) increased urine output c) protein in the urine d) fundal height at the level of umbilicus 25. Which of the following assessment findings indicates adverse reaction to Morphine Sulfate in a

gravida 5 para 5 client? a) elevated blood pressure b) increased respiratory rate c) boggy fundus of the uterus d) restlessness

26. A baby has been delivered 2 hours ago by a diabetic mother. The baby manifests high-pitched cry. The nurse should assess for which of the following conditions? a) fetal alcohol syndrome b) increased intracranial pressure c) prematurity d) hypoglycemia 27. Which of the following situations in a newborn necessitates urgent attention by the nurse? a) irregular respiratory patterns b) body temperature of 36.5 degree centigrade c) blood pressure of 65/41 mmHg d) meconium staining on the infant's body 28. A pregnant woman on 36 weeks gestation experiences sudden gush of fluids from the vagina. Which of the following should be the initial action by the nurse? a) notify the physician b) check the fluid pH c) prepare the client for delivery d) place the client in knee-to-chest position 29. The client with endometriosis is taking Danazol. Which of the following is the expected effect of the medication? a) it inhibits ovulation b) it relieves uterine spasm c) it reduces menstrual bleeding d) it prevents pregnancy 30. The nurse is giving health teachings to several pregnant clients. Which of the following statements of the clients should be given highest priority by the nurse? a) I enjoy working in the garden and keeping my hands dirty. It relaxes me b) I walk a mile every morning and 3 miles on weekends c) I watch the recipes on TV shows and cook them

d) I drive myself to work

31. Which of the following findings should the nurse report to the physician when observed in a 6month old infant? a) absent moro reflex b) positive kernig's sign c) positive babinski's sign d) absent tonic neck reflex 32. A pregnant client is admitted in the emergency room, with cervix fully dilated. Which of the following is the priority action by the RN to facilitate proper bearing down? a) put the client in sitting position with shoulders supported b) put the client in lithotomy position c) put the client in right side-lying position d) put the client in semi-sitting position and use elbows for support 33. The client is 3 days postpartum, and she bottle-feeds her newborn. She complains of hardness and swelling of hr breasts. Which of the following is the most essential intervention? a) apply ice cap over the breasts b) massage the breasts c) use breast pump to express the milk d) apply warm compress over the breasts 34. A woman is in active labor. In what position does the nurse properly place the client? a) semi-fowler's position b) side-lying position c) trendelenburg position supine position 35. The client has been diagnosed to have placenta previa. Which of the following should be included in the nursing care plan of the client? Select all that apply a) promote bed rest with bathroom privileges b) ask for prescription of internal fetal heart rate (FHR) monitoring c) perform vaginal examination every 8 hours d) place the client in the left lateral position e) administer blood transfusion as prescribed f) administer Rh globulin as prescribed if the mother is Rh negative g) prepare for premature delivery or cesarean section 36. Several female patients want to have Papanicolaou examination. Who among these patients should the nurse advise not to have the examination? The patient who states

a) the first day of my menstruation came this morning and I am bleeding profusely b) I am 21 years old, and have not had any sex at all c) I had the test 3 months ago and it was positive d) I have herpes simplex virus (HSV) and had sex 2 weeks ago 37. A mother who has just delivered a term baby, wants to delay breastfeeding for 3 days. What is the best interpretation of this mother's behavior? a) she has knowledge deficit regarding breastfeeding b) she doesn't want to breastfeed her child c) she doesn't want to have bonding with her child d) she doesn't want to accept her responsibility of caring for her child 38. A primigravid client at 8 weeks gestation tells the nurse that since having had sexual relations with a new partner 2 weeks ago, she has noticed flu-like symptoms, enlarged lymph nodes, and clusters of vesicles on her vagina. The nurse refers the client to a physician because the nurse suspects which of the following sexually transmitted diseases. a) gonorrhea b) chlamydia trachomatis c) syphilis d) herpes genitalis 39. A middle-aged woman has just returned from the recovery room after a right mastectomy. A top priority in planning her care is to minimize the pain she is experiencing. a) risk for ineffective airway clearance b) alteration in comfort c) potential for injury d) alteration in nutrition 40. A patient who is on her 39 weeks gestation comes to the hospital accopmpanied by her husband. She tells the nurse she thinks she is in labor. Which of the following questions should the nurse ask to help confirm if the patient is in true labor? a) do your contractions feel like severe menstrual cramps? b) do you feel pressure in your legs c) do you feel as if you can breathe easier? d) does your pain increase in intensity when you are moving around? 41. The client had been diagnosed to have PIH (pregnancy-induced hypertension). Which of the following should be included in her care? Select all that apply a) administer magnesium sulfate IV drip as ordered b) check urine for presence of protein c) have calcium gluconate readily available

d) monitor for elevated liver enzymes e) observe for elevated platelet counts f) encourage ambulation 42. A clinic nurse is teaching a pregnant client about the warning signs in pregnancy. Which of the following, if identified as a warning sign by the client would indicate that she understands the teaching? a) purplish discoloration of the vulva b) visual disturbances c) irregular, painless contractions d) urinary frequency 43. The breastfeeding mother of infant with lactose intolerance asks a nurse about dietary measures. The nurse tells the mother to avoid a) hard cheeses b) green leafy vegetables c) dried beans d) egg yolk 44. A newborn has a temperature of 35.8 C, pulse rate of 126/min, respiratory rate of 65/min. What action should the nurse take first? a) dry the newborn and wrap him with blanket b) put the newborn in a radiant warmer c) check the newborn's blood sugar level d) perform phototherapy to the newborn 45. The nurse on a night shift finds a multiparous patient 8 hours postpartum drenched in perspiration. The temperature is 99F, pulse is 66 bpm, and BP is 120/80 mmHg. Which of the following nursing diagnosis would be appropriate? a) risk for infection related to birth trauma b) ineffective thermoregulation related to hormonal changes c) ineffective tissue perfusion d) excess fluid volume related to normal postpartal diuresis 46. A mother complains of pain due to breast engorgement and is bottle-feeding her newborn. Which action by the mother needs follow-up? a) the mother applies warm compress on her breasts b) the mother wears support bra c) the mother applies cold compress on her breasts d) the mother takes prescribed analgesics 47. A nurse is assisting a client who is in first stage of labor (active phase). A priority nursing action

for the nurse is: a) encourage the woman to blow out strong, short breaths b) assist the client to a comfortable position in bed c) monitor client's temperature every hour d) evaluate fetal heart rate (FHR) every hour 48. A nurse on the obstetric unit is providing care to a woman in the active phase of the first stage of labor. Which statement if made by the mother should be a priority concern for the nurse? a) I will like to take a nap between contractions b) I have not voided in the last hour, although I feel I need to c) I am feeling some rectal pressure that is relieved when I push d) I am feeling contractions every 5 minutes 49. A nursing instructor asks a nursing student to describe the procedure for performing the Helmlich maneuver on an unconscious pregnant woman at 8 month's gestation. The student describes the procedure correctly if the student states that she or he will: a) place the hands in the pelvis to perform the thrusts b) perform abdominal thrusts until the object is dislodged c) perform left lateral abdominal thrusts until the object is dislodged d) place a rolled blanket under the right abdominal flank and hip area 50. A nursing student is assigned to a client in labor. A nursing instructor asks the student to describe fetal circulation, specially the ductus venosus. The nursing instructor determines that the student understands fetal circulation if the student states that the ductus venosus: a) connects the pulmonary artery to the aorta b) is an opening between the right and left atria c) connects the umbilical vein to the inferior vena cava d) connects the umbilical artery to the inferior vena cava 51. A nurse is performing an assessment of primipara who is being evaluated in clinic during her second trimester of pregnancy. Which of the following indicates an abnormal physical finding that necessities further testing? a) quickening b) braxton hicks contractions c) consistent increase in fundal height d) fetal heart rate of 180 bpm 52. A nurse is performing an assessment of a pregnant client who is at 28 weeks of gestation. The nurse measures the fundal height in centimeters and expects the finding to be which of the following? a) 22 cm

b) 30 cm c) 36 cm d) 40 cm 53. A nurse is reviewing the record of a client who has just been told that a pregnancy test is positive. The physician has documented the presence of Goodell's sign. The nurse determines that his sign indicates: a) a softening of the cervix b) the presence of fetal movement c) the presence of human chorionic gonadotrophin in the urine d) a soft blowing sound that corresponds to the maternal pulse during auscultation of the uterus. 54. A nurse is assisting in performing an assessment on a client who suspects that she is pregnant and is checking the client for probable signs of pregnancy. Select all probable signs of pregnancy. a) ballotment b) chadwick's sign c) uterine enlargement d) braxton hicks contractions e) outline of fetus via radiography or ultrasonography f) fetal heart rate detected by a nonelectronic device 55. A nurse is providing instructions regarding treatment of hemorrhoids to a client who is in the second trimester of pregnancy. Which statement by the client indicates a need for further teaching? a) I should avoid straining during bowel movements b) I can gently replace the hemorrhoids into the rectum c) I can apply ice packs to the hemorrhoids to reduce the swelling d) I should apply heat packs to the hemorrhoids to help the hemorrhoids shrink 56. A nurse has performed a nonstress test on a pregnant client and is reviewing the fetal monitor strip. The nurse interprets the test as reactive and understands that this indicates: a) normal findings b) abnormal findings c) the need for further evaluation d) that the findings on the monitor were difficult to interpret 57. A nonstress test is performed on a client who is pregnant and the results of the test indicate nonreactive findings. The physician orders a contraction stress test to be done and the results are documented as negative. The nurse interprets this finding as indicating: a) a normal test result b) an abnormal test result c) a high risk for fetal demise d) the need for a cesarean delivery

58. A nurse is reviewing a nutritional plan of care with a pregnant client and is identifying the food items highest in folic acid. The nurse determines that the client understands the foods that supply the highest amounts of folic acid if the client states that she will include which of the following in the daily diet? a) milk b) yogurt c) bananas d) leafy, green vegetables 59. A pregnant client tells a nurse that she has been craving "unusual foods." The nurse gathers additional assessment data from the client and discovers that the client has been ingesting daily amounts of white clay dirt from her backyard. Laboratory studies are performed on the client. The nurse reviews the laboratory results and determines that which of the following indicates a physiological consequence of this client's practice? a) hematocrit, 38% b) glucose, 86 mg/dL c) hemoglobin, 9.1 g/dL d) white blood cell count, 12,400 mm3 60. A pregnant client who is at 30 weeks gestation comes to the clinic for a routine visit, and the nurse performs an assessment on the client. Which observation made by the nurse during the assessment indicates need for teaching? a) the client is wearing sneakers b) the client is wearing knee-high hose c) the client is wearing flat shoes with rubber soles d) the client is wearing pants with an elastic waistband

61. A pregnant client visits a clinic for a scheduled prenatal appointment. The client tells the nurse that she frequently has a backache, and the nurse provides instructions regarding measures that will assist in relieving the backache. Which statement by the client indicates a need for further instructions? a) I should wear flat-heeled shoes b) I should sleep on firm mattress c) I should try to maintain good posture d) I should do more exercises to strengthen my back muscles 62. A clinic nurse has instructed a pregnant client in measures to prevent varicose veins during pregnancy. Which statement by the client indicates a need for further instructions? a) I should wear panty hose

b) I should wear support hose c) I should be wearing flat nonslip shoes that have good support d) I should wear knee-high hose as long as I don't leave them on longer than 8 hours 63. A clinic nurse is providing instructions to a pregnant client regarding measures that will assist in alleviating heartburn. Which statement by the client indicates an understanding of the instructions? a) I should avoid between-meal snacks b) I should lie down for an hour after eating c) I should use spices for cooking rather than using salt d) I should avoid eating foods that produce gas, such as beans, vegetables, and fatty foods like deep fried chicken 64. A nurse in a health care clinic is instructing a pregnant client about how to perform "kick counts." Which statement by the client indicates a need for further instructions? a) I will record the number of movements or kicks b) I need to lie flat on my back to perform the procedure c) a count of fewer than 10 kicks in a 12-hour period indicates the need to contact the physician d) I should place my hands on the largest part of my hands on the largest part of my abdomen and concentrate on the fetal movements to count the kicks 65. During a prenatal visit, the nurse is explaining dietary management to a client with diabetes mellitus. The nurse determines that the teaching has been effective if the client makes which statement? a) diet and insulin needs change during pregnancy b) I will plan my diet based on the results of urine glucose testing c) I will need to eat 600 more calories every day since I am pregnant d) I can continue with the same diet as before pregnancy, as long as it is well-balanced 66. A clinic nurse is performing a psychosocial assessment of a client who has been told that she is pregnant. Which assessment finding indicates to the nurse that the client is at high risk for contracting immunodeficiency virus (HIV)? a) a client who has a history of intravenous drug use b) a client who has a significant other who is heterosexual c) a client who has a history of sexually transmitted diseases d) a client who has had one sexual partner for the past 10 years 67. A nurse in maternity unit is providing emotional support to a client and her husband who are preparing to be discharged from the hospital after the birth of a dead fetus. Which statement, if made by the client, indicates a component of the normal grieving process? a) we want to attend a support group b) we never want to try to have a baby again c) we are going to try to adopt a child immediately

d) we are okay, and we are going to have another baby immediately 68. A nurse evaluates a hepatitis B-positive mother' ability for safe bottle-feeding of her infant during postpartum hospitalization. Which maternal action best exemplifies the mother's knowledge of potential disease transmission to the infant. a) the mother requests that the window be closed before feeding b) the mother holds the infant properly during feeding and burping c) the mother tests the temperature of the formula before initiating feeding d) the mother washes and dries her hands before and following self-care of the perineum and asks for a pair of gloves before feeding 69. A home care nurse visits a pregnant client who has a diagnosis of mild preeclampsia and who is being monitored for gestational hypertension. Which assessment finding indicates a worsening of the preeclampsia and the need to notify the physician? a) urinary output has increased b) dependent edema has resolved c) blood pressure reading is at the prenatal baseline d) the client complaints of headache and blurred vision 70. A client with a 38-week twin gestation is admitted to a birthing center in early labor. One of the fetuses is a breech presentation. Of the following interventions, which is the lowest priority in planning the nursing care of this client? a) measure fundal height b) attach electronic fetal monitoring c) prepare the client for possible cesarean section d) visually examine the perineum and vaginal opening 71.A nurse implements a teaching plan for a pregnant client who is newly diagnosed with gestational diabetes mellitus. Which statement, if made by the client, indicates a need for further teaching? a) I should stay on the diabetic diet b) I should perform glucose monitoring at home c) I should avoid exercise because of the negative effects on insulin production d) I should be aware of any infections and report signs of infection immediately to my health care provider 72. A client has just had surgery to deliver a nonviable fetus resulting from abruptio placentae. As a result of abruptio placentae, the client develops disseminated intravascular coagulation (DIC) and is told about the complication. The client begins to cry and screams, "God, just let me die now!" Which nursing diagnosis should direct care for this client at time? a) grieving related to the loss of the baby b) situational low self-esteem related to being ill c) deficient knowledge related to the disease process

d) hopelessness related to the loss of the baby and personal health 73. A pregnant client in the last trimester has been admitted to the hospital with a diagnosis of severe preeclampsia. A nurse monitors for complications associated with the diagnosis and assesses the client for: a) enlargement of the breast b) complaints of feeling hot when the room is cool c) periods of fetal movement followed by quiet periods d) evidence of bleeding, such as in the gums, petechia, and purpura 74. A nurse in a maternity unit is reviewing the records of the clients on the unit. Which client would the nurse identify as being at the greatest risk for developing disseminated intravascular coagulation (DIC)? a) a primigravida with mild preeclampsia b) a primigravida who delivered a 10-lb baby 3 hours ago c) a gravida II who has just been diagnosed with dead fetus syndrome d) a gravida IV who delivered 8 hours ago and has lost 500 ml of blood 75. A client in the first trimester of pregnancy arrives at a health care clinic and reports that she has been experiencing vaginal bleeding. A threatened abortion is suspected, and the nurse instructs the client regarding management of care. Which statement, if made by the client, indicates a need for further instructions. a) I will watch for the evidence of the passage of tissue b) I will maintain strict bed rest throughout the remainder of the pregnancy c) I will count the number of perineal pads used on a daily basis and note the amount and color of blood on the pad d) I will avoid sexual intercourse until the bleeding has stopped, and for 2 weeks following the last evidence of bleeding 76. A nurse is reviewing true and false labor signs with multiparous client. The nurse determines that the client understands the signs of true labor if she makes which statement? a) I won't be in labor until the baby engages b) my contractions will be felt in the abdominal area c) my contractions will not be as painful if I walk around d) my contractions will increase in duration and intensity 77. A client in labor has been pushing effectively for 1 hour. A nurse determines that the client,s primary physiological need at this time is to: a) ambulate b) rest between contractions c) change positions frequently

d) consume oral food and fluids

78. A nurse is caring for a client in labor. The nurse determines that the client is beginning the second stage of labor when which of the following assessments is noted? a) the contractions are regular b) the membranes have ruptured c) the cervix is dilated completely d) the client begins to expel clear vaginal fluids 79. A nurse is performing an assessment of a client who is scheduled for a cesarean delivery. Which assessment finding would indicate a need to contact the physician? a) hemoglobin of 11.0 g/dL b) fetal heart rate of 180 bpm c) maternal pulse rate of 85 bpm d) white blood cell count of 12,000/mm3 80. A nurse has provided discharge instructions to a client who delivered a healthy newborn infant by cesarean delivery. Which statement, if made by the client, indicates a need fro further instructions? a) I will begin abdominal exercises immediately b) I will notify the physician if I develop a fever c) I will turn on my side and push up with my arms to get out of bed d) I will lift nothing heavier than the newborn infant for at least two weeks 81. A nurse is caring for a client in labor and prepares to auscultate the fetal heart rate by using a Doppler ultrasound device. The nurse accurately determines that the fetal heart sounds are heard by: a) Noting whether the heart rate is greater than 140 bpm b) placing the diaphragm of the Doppler on the mother's abdomen c) palpating the maternal radial pulse while listening to the fetal heart rate d) Performing Leopold's maneuver first to determine the location of the fetal heart 82. A nurse is caring for a client in labor who is receiving oxytoxin (Pitocin) by intravenous infusion to stimulate uterine contractions. Which assessment finding would indicate to the nurse that the infusion needs to be discontinued? a) increased urinary output b) a fetal heart rate of 90 bpm c) three contractions occurring within a 10-minute period d) adequate resting tone of the uterus palpated between contractions

83. A nurse is reviewing the record of a client in the labor room and notes that the nurse-midwife has documented that the fetus is at negative 1 (-1) station. The nurse determines that the fetal presenting part is: a) 1 inch below the coccyx b) 1 inch below the iliac crest c) 1 cm above the ischial spine d) 1 fingerbreath below the symphysis pubis 84. nurse is monitoring a client in labor. The nurse suspects umbilical cord compression. If which of the following is noted on the external monitor tracing during a contraction? a) late decelerations b) early decelerations c) short-term variability d) variable decelerations 85. A labor and delivery room nurse has just received report on four clients. The nurse should assess which client first? a) a primiparous client in the active stage of labor b) a multiparous client who was admitted for induction of labor c) a client who is not contracting but has suspected premature rupture of the membranes d) a client who has just received an Iv loading dose of magnesium sulfate to stop preterm labor 86.A nurse is reviewing the physician's orders for a client admitted for premature rupture of membranes. Gestational age of the fetus is determined to be 37 weeks. Which physician's order should the nurse question? a) perform a vaginal examination every shift b) monitor maternal vital signs every 4 hours c) monitor fetal heart rate (FHR) continuously d) administer ampicillin 1gm as an intravenous piggyback (IVPB) every 6 hours 87. A nurse is providing emergency measures to a client in labor who has been diagnosed with a prolapsed cord. The mother becomes anxious and frightened and says to the nurse, "Why are all of these people in here? Is my baby going to be all right?" Which of the following nursing diagnosis would be most appropriate for this client at this time? a) fear b) fatigue c) powerlessness d) ineffective coping 88. A nurse in the postpartum unit is caring for a client who has just delivered a newborn infant following a pregnancy with a placenta previa. The nurse reviews the plan of care and prepares to monitor the client for which of the following risks associated with placenta previa?

a) infection b) hemorrhage c) chronic hypertension d) disseminated intravascular coagulation 89. A maternity nurse is caring for a client with abruptio placentae and is monitoring the client for disseminated intravascular coagulopathy. Which assessment finding is least likely to be associated with DIC? a) prolonged clotting times b) decreased platelet count c) swelling of the calf of one leg d) petechiae, oozing from injection sites, and hematuria 90. A nurse is assessing a pregnant client in the second trimester of pregnancy who was admitted to the maternity unit with a suspected diagnosis of abruptio placentae. Which of the following assessment findings would the nurse expect to note if this condition is present? a) a soft abdomen b) uterine tenderness c) absence of abdominal pain d) painless, bright red vaginal bleeding 91. A nurse in labor room is assisting with the vaginal delivery of a newborn infant. The nurse would monitor the client closely for risk of uterine rupture if which of the following occurred? a) forcep delivery b) schultz presentation c) hypotonic contractions d) weak bearing-down efforts 92. A clinic nurse is performing a prenatal assessment on a pregnant client. The nurse would implement teaching related to the risk of abruptio placentae if which of the following information was obtained on assessment? a) the client is 28 years of age b) this is the second pregnancy c) the client has a history of hypertension d) the client performs moderate exercise on a regular daily schedule 93. A nurse is performing an initial assessment on a client who has just been told that pregnancy test is positive. Which assessment finding would indicate that the client is at risk of preterm labor? a) the client is a 35-year old primigravida b) the client has history of cardiac disease c) the client's hemoglobin level is 13.5 g/dL d) the client is a 20-year old primigravida of average weight and height

94. A nurse in labor room is monitoring a client with dysfunctional labor for signs of fetal or maternal compromise. Which of the following assessment findings would alert the nurse to compromise? a) maternal fatigue b) coordinated uterine contractions c) progressive changes in the cervix d) persistent nonreassuring fetal heart rate 95. A nurse is assigned to care for a client with hypotonic uterine dysfunction and signs of slowing labor. The nurse is reviewing the physician's orders and would expect to note which of the following prescribed treatments for this condition? a) increased hydration b) oxytocin (Pitoxin) infusion c) medication that will provide sedation d) administration of a tocolytic medication 96. A nurse is performing an assessment on a client diagnosed with placenta previa. Which of these assessmentfindings would the nurse expect to note? Select all that apply a) uterine rigidity b) uterine tenderness c) severe abdominal pain d) bright red vaginal bleeding e) soft, relaxed, nontender uterus f) fundal height may be greater than expected fro gestational age 97. A nurse is caring for four 1-day postpartum clients. Which client has an abnormal finding that would require further intervention? a) the client with mild after pains b) the client with a pulse rate of 60 bpm c) the client with colostrum discharge from both breast d) the client with lochia that is red and has foul-smelling odor 98. A nursing student is preparing to perform a cardiovascular assessment on a postpartum client. A nursing instructor asks the student about the procedure to elicit Homan's sign. Which response by the nursing student would indicate an understanding of this assessment technique? a) I will ask the client to raise her legs up to her waist and then to lower her legs slowly b) I will ask the client to raise her legs and to try to lower them against pressure from my hand c) I will ask the client to extend her legs flat on the bed, and I will grasp her foot gently dorsiflex it forward d) I will ask the client to extend her legs flat on the bed, and I will grasp her foot and sharply extend it backward

99. A nurse is planning care for a postpartum client who had a vaginal delivery 2 hours ago. The client had a midline episiotomy and has several hemorrhoids. What is the priority nursing diagnosis for this client? a) acute pain b) disturbed body image c) impaired urinary elimination d) risk for imbalanced fluid volume 100. A nurse is providing postpartum instructions to a client who will be breast-feeding her newborn. The nurse determines that the client has understood the instructions if she makes which of the following statements? Select all that apply. a) I will use soap to wash my breasts often b) drinking alcohol can affect my milk supply c) the use of caffeine can decrease my milk supply d) I will start my estrogen birth control pills again as soon as I get home e) I know if my breasts get engorged I will limit my breast-feeding and supplement the baby f) I plan on having bottled water available in the refrigerator so I can get additional fluids easily

You might also like